LSAT and Law School Admissions Forum

Get expert LSAT preparation and law school admissions advice from PowerScore Test Preparation.

User avatar
 Dave Killoran
PowerScore Staff
  • PowerScore Staff
  • Posts: 5853
  • Joined: Mar 25, 2011
|
#94665
Complete Question Explanation
(The complete setup for this game can be found here: lsat/viewtopic.php?f=280&t=8537)

The correct answer choice is (A)

As discussed during the setup, one of the novel distributions allows for three French novels and no Russian novels to be selected. Hence, answer choice (A) is correct.

Answer choice (B) is incorrect because selecting only one French novel would result in fewer than five works being selected, a violation of the condition in the game scenario.

Answer choice (C) is incorrect because the fourth rule disallows all three plays from being selected.

Answer choice (D) is incorrect because from the third rule all three French novels would have to be selected, which would result in a violation of the second rule that a maximum of four novels are selected.

Answer choice (E) is incorrect because this selection would violate the first rule, which limits the number of French works to four.
 Basia W
  • Posts: 108
  • Joined: Jun 19, 2014
|
#16549
Good evening,

I just wanted to make sure that I understood this rule properly: "at least as many French novels as Russian novels are selected"

8) In this answer choice however, it is said that it is possible for "no Russian novels to be selected." If a minimum of three novels are to be selected and there are no Russian novels wouldn't that mean that there were no French novels either violating this rule?

Thank you for clarifying,

Best,

Basia
 Robert Carroll
PowerScore Staff
  • PowerScore Staff
  • Posts: 1787
  • Joined: Dec 06, 2013
|
#16563
Basia,

The rule means that the number of Russian novels cannot exceed the number of French novels. The number of French novels can exceed the number of Russian novels, or they can be equal:

FN > or = RN

You may have it backwards! I hope this has helped.

Robert Carroll
 Basia W
  • Posts: 108
  • Joined: Jun 19, 2014
|
#16576
Hello,

yes that explains it- thank you!

Best,

Basia

Get the most out of your LSAT Prep Plus subscription.

Analyze and track your performance with our Testing and Analytics Package.